LSAT and Law School Admissions Forum

Get expert LSAT preparation and law school admissions advice from PowerScore Test Preparation.

User avatar
 Dave Killoran
PowerScore Staff
  • PowerScore Staff
  • Posts: 5852
  • Joined: Mar 25, 2011
|
#88028
Complete Question Explanation
(The complete setup for this game can be found here: lsat/viewtopic.php?f=164&p=88024#p88024)

The correct answer choice is (C).

With S preparing an opening argument, and L and R/G preparing an opening argument, our initial diagram for this question appears as follows:

G1-Q6-d1.png

Some students stop here, but there is more to the setup for this question. From the first rule, M must be paired with G or V. If R were to prepare an opening statement, then G, V, and M would each have to prepare final statements, and thus there would be no way to meet the conditions stated in the first rule. Thus, we can conclude that R cannot prepare an opening statement, and G must prepare an opening statement:

G1-Q6-d2.png

M, R, and V must now prepare final statements, and in order to satisfy the first rule, M must be teamed with G, leaving R and V to rotate between S and L:

G1-Q6-d3.png

Accordingly, answer choice (C) is correct.
You do not have the required permissions to view the files attached to this post.

Get the most out of your LSAT Prep Plus subscription.

Analyze and track your performance with our Testing and Analytics Package.